Q24

User avatar
 
LSAT-Chang
Thanks Received: 38
Atticus Finch
Atticus Finch
 
Posts: 479
Joined: June 03rd, 2011
 
 
trophy
Most Thankful
trophy
First Responder
 

Q24

by LSAT-Chang Thu Sep 15, 2011 6:50 pm

What is wrong with A? I was down to A and E and ultimately went with A because I thought it was more reasonable.. but is it wrong because we have no evidence whether these systems have met the original expectations of computer specialists?
User avatar
 
ohthatpatrick
Thanks Received: 3808
Atticus Finch
Atticus Finch
 
Posts: 4661
Joined: April 01st, 2011
 
This post thanked 3 times.
 
 

Re: Q24

by ohthatpatrick Fri Sep 16, 2011 1:18 pm

You hit the nail on the head.

In (A), we have no proof that 'these systems have met the original expectations of computer specialists'.

Perhaps the closest match in the passage that we have for 'original expectations' is line 10, but it says that these systems have 'fallen short of optimistic early predictions'.

To prove (E), we could use 33-37 and 54-57. The fact that we need to exceed the 'capabilities of the present or the foreseeable future' and to develop a system that moves us past an 'apparently intractable problem' is a good enough match for 'we need a breakthrough'.

A couple quick thoughts:

1. When the question stem says "most likely agree", treat it like "what is inferred/implied/suggested" ... i.e., figure out what's wrong with 4 answers, and find a line reference or two to convince you of the correct answer.

2. LSAT loves to create trap answers that are 1/2 right, 1/2 wrong. If your nature is to read answer choices looking for what works, you're liable to be fished in by the 1/2 right portion. Instead, train your brain to read answer choices looking for what's wrong ... here, your correct analysis that we never established these systems met anyone's original expectations would have forced you to eliminate A.
User avatar
 
LSAT-Chang
Thanks Received: 38
Atticus Finch
Atticus Finch
 
Posts: 479
Joined: June 03rd, 2011
 
 
trophy
Most Thankful
trophy
First Responder
 

Re: Q24

by LSAT-Chang Fri Sep 16, 2011 1:43 pm

ohthatpatrick Wrote:2. LSAT loves to create trap answers that are 1/2 right, 1/2 wrong. If your nature is to read answer choices looking for what works, you're liable to be fished in by the 1/2 right portion. Instead, train your brain to read answer choices looking for what's wrong ... here, your correct analysis that we never established these systems met anyone's original expectations would have forced you to eliminate A.


Thank you for the great advices! I try really hard not to "search" for the correct answer, but I think I have a tendency to fall into those 1/2 right, 1/2 wrong answers (exactly why I wasn't able to eliminate A) -- it is as if my brain automatically "makes" the other 1/2 wrong answer merge into the 1/2 right answer and makes it "sound" right :shock: I was tempted to make myself "believe" that the first part was true as well since the latter half was 100% provable based on text. I definitely have to watch out for these types of answers! Thanks again!!
 
myradin
Thanks Received: 0
Vinny Gambini
Vinny Gambini
 
Posts: 6
Joined: November 26th, 2012
 
 
 

Re: Q24

by myradin Wed Jan 09, 2013 5:58 pm

I see how E is directly supported. Can anyone give a good reason that C was a bad choice? It was my original answer and I thought pretty inferable...
 
nflamel69
Thanks Received: 16
Atticus Finch
Atticus Finch
 
Posts: 162
Joined: February 07th, 2011
 
 
 

Re: Q24

by nflamel69 Wed Apr 10, 2013 7:42 pm

The answer distinguishes between legal aid and legal research tools. But does anywhere in the passage author make such distinctions? I think the closest inference you can make is to say the author think the current A.I. legal reasoning system is inadequate, but not what they most likely to be used as.
 
ganbayou
Thanks Received: 0
Atticus Finch
Atticus Finch
 
Posts: 213
Joined: June 13th, 2015
 
 
 

Re: Q24

by ganbayou Fri Jul 10, 2015 11:32 pm

Why is C wrong?
I thought since the author seems providing reasons why the systems would not work, I thought the author thinks it should be used as just a tool instead of analyzing something.
Did I make too much inference that might not be true?

Thanks,
User avatar
 
ohthatpatrick
Thanks Received: 3808
Atticus Finch
Atticus Finch
 
Posts: 4661
Joined: April 01st, 2011
 
 
 

Re: Q24

by ohthatpatrick Wed Jul 15, 2015 7:52 pm

The previous poster's explanation of why (C) is wrong is great!

Your notion is completely reasonable, but also completely fabricated. Where's your line reference? When did the author mention using these systems as "research tools"? When did he put a wedge into which is more likely: using it as research tool or aid in analysis?

Make sure you're not using the everyday person's conversational understanding of "inferred/implies/suggests". These questions don't allow us to speculate.

They are still just a question of "Which of these can you prove, or best support, from the text?"

So until you have a line reference to support (C), you can't think about picking it. Make sure you're not attempting to answer these from your gut.

== wrong answers ==

(A) Where did it say it met the original expectations?

(B) Where did the author say that "inaccessibility of a sufficient quantity of legal documents" is the PRIMARY reason for these systems' disappointing progress?

(C) Where does the author say these systems will more likely be research tools?

(D) Where does the author say that rule systems will replace case-based systems?


Correct answer (E) is supported by lines 32-36.
 
lsat2016
Thanks Received: 0
Jackie Chiles
Jackie Chiles
 
Posts: 28
Joined: June 18th, 2015
 
 
 

Re: Q24

by lsat2016 Wed Dec 16, 2015 3:20 am

ohthatpatrick Wrote:The previous poster's explanation of why (C) is wrong is great!

Your notion is completely reasonable, but also completely fabricated. Where's your line reference? When did the author mention using these systems as "research tools"? When did he put a wedge into which is more likely: using it as research tool or aid in analysis?

Make sure you're not using the everyday person's conversational understanding of "inferred/implies/suggests". These questions don't allow us to speculate.

They are still just a question of "Which of these can you prove, or best support, from the text?"

So until you have a line reference to support (C), you can't think about picking it. Make sure you're not attempting to answer these from your gut.

== wrong answers ==

(A) Where did it say it met the original expectations?

(B) Where did the author say that "inaccessibility of a sufficient quantity of legal documents" is the PRIMARY reason for these systems' disappointing progress?

(C) Where does the author say these systems will more likely be research tools?

(D) Where does the author say that rule systems will replace case-based systems?


Correct answer (E) is supported by lines 32-36.


Hi,
Doesn't the first sentence "computers have long been utilized in the sphere of law in the form of word processors, spreadsheets, LEGAL RESEARCH SYSTEMS, and practice management systems" serve as evidence for C?
 
YihanX379
Thanks Received: 0
Vinny Gambini
Vinny Gambini
 
Posts: 4
Joined: August 04th, 2017
 
 
 

Re: Q24

by YihanX379 Tue Sep 12, 2017 11:58 am

I also think the first sentence of the passage serves as a supporting reference of answer (C)...
 
egonza14
Thanks Received: 2
Vinny Gambini
Vinny Gambini
 
Posts: 7
Joined: May 14th, 2015
 
 
 

Re: Q24

by egonza14 Wed Jan 24, 2018 4:59 pm

I was between C) and E) as well, but I eliminated C because I felt the "research tools" component was bait.

The author mentions research tools only in the beginning of the passage, and the author refers to "Computers" being used in the sphere of law in the form of "word processors, spreadsheets, legal research systems".

Nowhere in the passage, does the author mention using Legal Reasoning Systems as research aids. We only know that they currently aren't great tools for legal analysis. Nothing about research. Furthermore, the author never makes any relative comparison between Research v. Analysis.

So by POE, E) looked good to me, thanks to the author pointing out the current challenges that the systems experience.
 
egonza14
Thanks Received: 2
Vinny Gambini
Vinny Gambini
 
Posts: 7
Joined: May 14th, 2015
 
This post thanked 2 times.
 
 

Re: Q24

by egonza14 Wed Jan 24, 2018 5:12 pm

ALSO, if you still have trouble with choosing between C and E here, check out the explanation for Q28 of this passage. A lot of people were stuck between B and C in that question, which played upon the same bait of "computers/computer systems" vs "legal reasoning systems."